Skip to Main Content

PrepTest 78, Game 3, Question 16



This is number 16, which is the fifth question in PT 78, Game 3. It's a global question asking which of the following could be true. So our only option here is to go to the answers and see whether it could be true based on our rules. Answer choice A says M is second, but M cannot be second because remember M has to come before S, that was one of our rules, and S can't go first.

So if you put the M in 2, you're gonna force the S into 1, that's a contradiction, A can't be true. Answer choice B says L is second, and it looks like that might be able to happen in this world over here. It couldn't happen in the first world because if L were second, then you'd have to put S first which contradicts the first rule.

But it turns out even if you try to put L second in the second of our OR options L would go over here. That means since S can't go first, H would have to go first. And now we've triggered our conditional rule. If H is before L, then you have to see M before L. But when L is second and H is first, there's no room to put the M, that's why this doesn't work.

Answer choice C says V is second, but you can get rid of that for the same reason A was wrong. V has to come after S. And so if you tried to put V in 2, S would be in 1 which contradicts the first rule. And so this doesn't work. Answer choice D says L is third.

Now this is a difficult one. It seems like it might be able to work in either of our OR possibilities here. Let's start with the first one. If you put L in three, then what's gonna happen is you have to put the S going first, which doesn't work. The reason S would have to go first is because S is before every other remaining letter.

So if you put L in three, S would be going in first, and that contradicts the first rule. So it doesn't work in this world. But what if were in the second of our OR options. If L is third, then the H would have to go first because S can't go first. And then now we have the second rule trigger, right?

If H is before L, then that means M has to be before L. But now we run into a problem because if H is before l like this, the only place for M to go before L is in the second spot. But now we're not gonna be able to accommodate S before M. So that's why putting L in third does not work in either of our OR options, it is not a possibility.

Through process of elimination, the answer would have to be E. In a time situation, if we prove the other four wrong, it's totally fine to just move on. But since this is review, let's figure out why M could be 5. Let's see, could we put the M, 5? Well, it looks like M could go 5 in the first of our OR options.

You would put l first, you could put S second, and then you could put V, T, M in 5, H in 6, this is something that works, and so that's why E could be true.

Read full transcript